QUESTION TEXT: Consultant: If Whalley sticks with her current…
QUESTION TYPE: Sufficient Assumption
CONCLUSION: Whalley will win if she keeps using her current strategy.
REASONING: Her current strategy leads to her losing slightly with voters under 50, and winning by a larger amount with voters above 50.
ANALYSIS: This question is almost a necessary assumption question. It’s also a hidden math question.
The stimulus tells us about Whalley’s performance with both groups, but not how big those groups are. If there are far more voters under 50, Whalley will lose. If there are equal voters in both groups, or more over 50, then Whalley wins.
So if we assume there are equal or more voters above 50, then the conclusion is true.
___________
- This is phrased like a necessary assumption answer. This answer shows Whalley couldn’t improve her strategy by looking for under-50 voters. But it doesn’t show Whalley’s current strategy will win.
- This explains why Whalley’s strategy produces different results in both groups. But it doesn’t show Whalley will win.
- This tells us what happens if Whalley changes. But we’re trying to prove that Whalley will win with her current strategy.
- CORRECT. If this is true, Whalley will win. The group she is winning is larger, and she’s winning it by a larger amount than she is losing the under 50 vote. So she will have a larger number of votes.
- This is irrelevant. We’re trying to prove that Whalley’s current platform will win.

Free Logical Reasoning lesson
Get a free sample of the Logical Reasoning Mastery Seminar. Learn tips for solving LR questions
Leave a Reply